Finding Values of p for Convergence: (-1)^(n-1)/ n^p

  • Thread starter Thread starter remaan
  • Start date Start date
  • Tags Tags
    Convergence
Click For Summary
SUMMARY

The convergence of the series defined by (-1)^(n-1) / n^p is determined by the value of p. For the series to converge, p must be greater than 1, aligning with the properties of p-series. The discussion clarifies that while the series has alternating terms, it is essential to recognize that it behaves like a p-series in terms of convergence criteria. The confusion regarding the series classification was addressed, emphasizing the need for clear communication in mathematical reasoning.

PREREQUISITES
  • Understanding of series convergence criteria
  • Familiarity with p-series and their properties
  • Basic knowledge of alternating series
  • Ability to analyze mathematical expressions and notation
NEXT STEPS
  • Study the properties of alternating series and the Alternating Series Test
  • Learn about the implications of the p-series test for convergence
  • Explore examples of series with varying p values to observe convergence behavior
  • Investigate the relationship between series convergence and integral tests
USEFUL FOR

Students studying calculus, mathematicians interested in series convergence, and educators teaching series analysis in higher education.

remaan
Messages
132
Reaction score
0

Homework Statement


For what values of p is the series con.
(-1)^ (n-1)/ (n)^(p)


Homework Equations



In order for a p series to converge, p should be greater than 1

The Attempt at a Solution



So an = 1/ n^p

and p > 1

Is that right ?
 
Physics news on Phys.org
You're making so many abbreviations and omissions that it's hard to tell what you're thinking -- I think you even confused yourself because of it. Try again, but this time say what you're doing, and why.
 
I don't think so,, the only abbreviation is Con. which means Convergent

** Am trying to find the valu of p for which the series will converge

What I thought about is : becauce it 's a p-series , then p should be P greater than 1 ?

What do you think ??
 
remaan said:
becauce it 's a p-series
No it's not.
 
So, give a hint? !
 
Oh, you I mean this part is a p- series, the one I considered as an
 
Question: A clock's minute hand has length 4 and its hour hand has length 3. What is the distance between the tips at the moment when it is increasing most rapidly?(Putnam Exam Question) Answer: Making assumption that both the hands moves at constant angular velocities, the answer is ## \sqrt{7} .## But don't you think this assumption is somewhat doubtful and wrong?

Similar threads

  • · Replies 7 ·
Replies
7
Views
2K
  • · Replies 5 ·
Replies
5
Views
2K
  • · Replies 14 ·
Replies
14
Views
2K
  • · Replies 2 ·
Replies
2
Views
1K
  • · Replies 1 ·
Replies
1
Views
1K
  • · Replies 2 ·
Replies
2
Views
2K
  • · Replies 1 ·
Replies
1
Views
2K
Replies
8
Views
2K
  • · Replies 15 ·
Replies
15
Views
2K
  • · Replies 5 ·
Replies
5
Views
2K